HW02 Solutions-Calc408K UT

download HW02 Solutions-Calc408K UT

of 11

Transcript of HW02 Solutions-Calc408K UT

  • 8/9/2019 HW02 Solutions-Calc408K UT

    1/11

    lin (ll25879) HW02 schultz (55325) 1

    This print-out should have 21 questions.Multiple-choice questions may continue onthe next column or page nd all choicesbefore answering.

    001 10.0 points

    If x 1, x 2 are the solutions of the equation

    73x2

    = 1493x+1

    ,

    compute the value of |x 1 x 2|.

    1. |x 1 x 2| = 33

    2.

    |x 1

    x 2

    |=

    2 3

    3

    3. |x 1 x 2| = 2 3

    3 correct

    4. |x 1 x 2| = 2

    5. |x 1 x 2| = 2Explanation:

    Since1493x+1 = 7

    6x2,

    the equation can be written as

    73x2

    = 7 6x2,

    which in turn can be rewritten as

    3x 2 = 6x 2by taking logs to the base 7 of both sides. Bythe quadratic formula, therefore,

    x 1, x 2 = 3 33 .Thus

    |x 1 x 2| = 2 3

    3 .

    002 10.0 points

    Find y when

    2x = 32 y, 32x = 16 y+1 .

    1. y = 421

    correct

    2. y = 221

    3. y = 27

    4. y = 17

    5. y = 5

    21Explanation:

    The equations

    2x = 32 y, 32x = 16 y+1

    are equivalent to

    2x = 2 5y, 25x = 2 4(y+1) .

    Now set v = 2x

    and u = 2y. Then

    v = u5, v5 = 16 u 4 .

    Substituting for v in the second equation wethus get

    u 25 = 16 u 4, i .e ., u = 2 421 .

    Consequently, since u = 2 y,

    y = 421

    .

    003 10.0 points

    Which of the following is the graph of

    f (x ) = 2 2x1 ?

  • 8/9/2019 HW02 Solutions-Calc408K UT

    2/11

    lin (ll25879) HW02 schultz (55325) 2

    1.

    2 424

    2

    4

    2

    4

    2. 2 424

    2

    4

    2

    4

    3.2 424

    2

    4

    2

    4

    4.2 424

    2

    4

    2

    4

    5.2 424

    2

    4

    24

    correct

    6.2 424

    2

    4

    24

    Explanation:Since

    limx

    2x = 0 ,

    we see that

    limx

    f (x ) = 2 ,

    in particular, f has a horizontal asymptotey = 2. This eliminates all but two of thegraphs. On the other hand, f (0) =

    32

    , so thy-intercept of the given graph must occur aty =

    32

    .Consequently, the graph is of f is

    2 424

    2

    4

    2

    4

    004 10.0 points

  • 8/9/2019 HW02 Solutions-Calc408K UT

    3/11

    lin (ll25879) HW02 schultz (55325) 3

    Suppose f is a 1-1 function with domain Aand range B.

    What is the range of f 1?

    1. Cant be determined.

    2. B3. A correct

    Explanation:The domain of f is the range of its inverse

    f 1 , while the domain of f 1 is the range of f .

    005 10.0 points

    If the graph of f is

    4 8

    4

    8

    4

    8

    4

    8

    which of the following is the graph of f 1(x )?

    1.

    4 848

    4

    8

    4

    8

    2.

    4 848

    4

    8

    48

    3.

    4 848

    4

    8

    4

    8

    4.

    4 8484

    8

    4

    8

    5.

    4 848

    4

    8

    4

    8

  • 8/9/2019 HW02 Solutions-Calc408K UT

    4/11

    lin (ll25879) HW02 schultz (55325) 4

    6.

    4 848

    4

    8

    48

    correct

    Explanation:The graph of f 1(x ) is obtained by reect-

    ing the graph of f over the line y = x:

    4 8484

    8

    4

    8

    006 10.0 points

    If f 1 is the inverse of f , determine thevalue of f (f 1(5)).

    1. f (f 1(5)) = 25

    2. f (f 1(5)) = 125

    3. Need to know f

    4. f (f 1(5)) =

    15

    5. f (f 1(5)) = 5 correct

    Explanation:The inverse, f 1, of f has the property that

    f (f 1(x )) = x, f 1(f (x )) = x .Consequently,

    f (f 1(5)) = 5 .

    007 10.0 points

    Determine the inverse function, f 1, of fwhen

    f (x ) = 5 + 4 x , x

    5

    4 .

    1. f 1(x ) = x2 + 4

    5 , x 0

    2. f 1(x ) = x2 4

    5 , x

    54

    3. f 1(x ) = 5 x 2

    4 , x 0

    4. f 1(x ) = x2 5

    4 , x 0 correct

    5. f 1(x ) = 4 x 2

    5 , x

    54

    6. f 1(x ) = x2 5

    4 , x

    45

    Explanation:

    Since f is dened on 54 , and is increasing on this interval, the inverse function,

    f 1, exists and has range 54

    , ; further-more, since f has range [0 , ), the inverse of has domain [0 , ).To determine f 1 we rst solve for x in

    y = 5 + 4 x ,

    and then interchange x, y . Solving rst for we see that

    4x = y2 5 .Consequently, the inverse of f is dened on[0, ) by

    f 1(x ) = x2 5

    4 .

  • 8/9/2019 HW02 Solutions-Calc408K UT

    5/11

    lin (ll25879) HW02 schultz (55325) 5

    008 10.0 points

    Find the inverse of

    f (x ) = 1 + 3x9 2x

    .

    1. f 1(x ) = 9x 12x + 3

    correct

    2. f 1(x ) = 9x 13x + 2

    3. f 1(x ) = 9x + 12x + 3

    4. f 1(x ) = 3x 12x + 9

    5. f 1(x ) = 9x + 13x + 2

    Explanation:To determine the inverse of f we rst solve

    for x iny =

    1 + 3x9 2x

    .

    In this case

    x = 9y 12y + 3

    .

    The inverse f 1 is now obtained by inter-changing x, y . Thus

    y = f 1(x ) = 9x 12x + 3

    .

    009 10.0 points

    Rewrite

    5log3 x = 3in equivalent exponential form.

    1. x5 = 27

    2. x5 = 10

    3. x3 = 127

    4. x3 = 10

    5. x5 = 127

    correct

    Explanation:By exponentiation to the base 3,

    35log3 x = 127

    .

    But

    35log3 x = 3 log3 x5

    = x5.

    Hence the exponential form of the given equa-tion is

    x5

    = 127 .

    keywords: LogFunc, LogFuncExam,

    010 10.0 points

    Use properties of logs to simplify the ex-pression

    log7(x x 2 63 ) + log 7(x + x 2 63 ).1. log9 7

    2. 1 + log9 7

    3. 7 + log7 9

    4. 1 + log7 9 correct

    5. log7 9

    Explanation:By properties of logs the given expressioncan be rewritten as

    log7 (x x 2 63)(x + x 2 63 )= log 7 x 2 x 2 63

    2 = log 7 63

    Butlog7 63 = log7 7 + log7 9 .

  • 8/9/2019 HW02 Solutions-Calc408K UT

    6/11

    lin (ll25879) HW02 schultz (55325) 6

    Thus the given expression reduces to

    1 + log 7 9 .

    keywords: LogFunc, LogFuncExam,

    011 10.0 points

    Find the value of x when

    x = 23

    4log4 4 7 7 log7 4

    without using a calculator.

    1. x = 1112

    correct

    2. x = 1

    3. x = 1

    4. x = 1312

    5. x = 1112

    Explanation:Since

    4log4 x = x, 7 log7 x = 1x

    ,

    we see that

    4log4 4 = = 4 ,

    while7 log7 4 = =

    14

    .

    Consequently,

    x = 83

    74

    = 1112

    .

    012 10.0 points

    Find the value of x when

    x = 3log 6 4 63 6log3

    3 34 .

    1. x = 7112

    2. x = 356

    3. x = 64. x =

    234

    correct

    5. x = 173

    Explanation:Since

    r loga z = log a zr , loga a = 1 ,

    we see that

    log6 4 63 = 3

    4 ,

    whilelog3

    3 34 = 43

    .

    Consequently,

    x = 234 .

    013 10.0 points

    Use properties of logs to simplify the ex-pression

    loga 16 + 25

    loga 8 25

    loga 4 + loga1

    275

    as much as possible.

    1. loga 8 correct

    2. loga 25

    3. 8

    4. loga 16

  • 8/9/2019 HW02 Solutions-Calc408K UT

    7/11

    lin (ll25879) HW02 schultz (55325) 7

    5. loga 32

    Explanation:By properties of logs the given expression

    can be rewritten as

    loga 24 8 254

    25 2 2

    25

    = log a 8 .

    014 10.0 points

    Simplify the expression

    f (x ) = e6(ln6) log 6 x

    as much as possible.

    1. f (x ) = e6

    2. f (x ) = x6 correct

    3. f (x ) = e36

    4. f (x ) = x36

    5. f (x ) = 6 x

    Explanation:By the laws of exponents,

    f (x ) = e6(ln 6)log 6 x = ( e6ln6 )log6 x .

    So by using the identities

    eln y = y , 6log6 y = y

    we see that

    e6ln6 = eln 66

    = 6 6 .

    Consequently,

    f (x ) = 6 6log6 x = 6 log6 x6

    = x6 .

    015 10.0 pointsUse the properties of logarithms to expand

    ln a (b4 + c3) .1. 2 ln a + 2ln b4 + c3

    2. 2 ln a 2 ln b4 + c3

    3. ln a

    2 +

    ln(4b + 3 c)2

    4. ln a

    2 +

    ln b4 + c3

    2 correct

    5. ln a + ln b4 + c3

    2

    Explanation:

    ln a (b4 + c3) = ln a b4 + c31/ 2

    = 12

    ln a b4 + c3

    = 12

    ln a + ln( b4 + c3)

    = 12

    ln a + 12

    ln(b4 + c3)

    016 10.0 points

    Which one of the following could be thegraph of

    f (x ) = log2(3 x )

    when dashed lines indicates asymptotes?

    1.

    2 4

    2

    2

  • 8/9/2019 HW02 Solutions-Calc408K UT

    8/11

    lin (ll25879) HW02 schultz (55325) 8

    2.

    2 4

    2

    2

    correct

    3.

    2 4

    2

    2

    4.

    2 4

    2

    2

    5.

    2 4

    2

    2

    6.

    2 4

    2

    2

    Explanation:Lets rst review some properties of log 2(x

    and log2(x ). Since log2(1) = 0, the graphof log2(x ) has x-intercept at x = 1. On theother hand, log 2(x ) is dened only on (0 , and

    limx 0

    +log2(x ) = , limx log2(x ) = ,

    so x = 0 is a vertical asymptote. Thus thegraph of log2(x ) is

    2 4

    2

    2

    To get the graph of log2(x ) we simply ipthe one for log2(x ) over both the x-axis andthe y-axis, producing

  • 8/9/2019 HW02 Solutions-Calc408K UT

    9/11

    lin (ll25879) HW02 schultz (55325) 9

    24

    2

    2

    To obtain the graph of y = log2(3 x )from this last one all we have to do now istranslate horizontally to the right by 3, pro-ducing

    2 4

    2

    2

    keywords: LogFunc, LogFuncExam,

    017 10.0 points

    Solve for x when

    e2x = 116

    .

    1. x = 8 ln4

    2. x = 4 ln4

    3. x = 2 ln2

    4. x = 2ln2 correct

    5. x = 8ln26. x = 4ln4

    Explanation:

    Taking logs of both sides we see that

    ln(e2x ) = ln 116

    .

    Thusln

    116

    = 2x ln e = 2x .

    Now

    ln 116

    = ln2 4 = 4ln2 .

    Consequently,

    x = 2ln2 .

    keywords: LogFunc, LogFuncExam,

    018 10.0 points

    Determine if

    f (x ) = ln(5 x + 2) , x > 25 ,has an inverse, and if it does, nd this inverse.

    1. f 1(x ) = ex 2

    5 , x >

    25

    2. f 1(x ) = 1

    ln(5x + 2) , x >

    25

    3. f 1(x ) = 1

    ln(5x

    2)

    , x > 25

    4. f 1(x ) = ex 2

    5 , < x < cor-

    rect

    5. f 1(x ) = ex + 2

    5 , < x <

    6. f 1(x ) does not exist

    Explanation:

  • 8/9/2019 HW02 Solutions-Calc408K UT

    10/11

    lin (ll25879) HW02 schultz (55325) 10

    Since

    ddx

    [ln(5x + 2)] = 5

    5x + 2 > 0 , x >

    25

    ,

    f is strictly increasing on 25

    , , so f hasan inverse.

    To determine this inverse we solve for y in

    x = ln(5 y + 2) .

    But then

    5y + 2 = ex , i .e ., y = ex 2

    5 ,

    so

    f 1

    (x ) =

    ex

    2

    5 .

    019 10.0 points

    Find the inverse function of

    f (x ) = e4 x , x 0 .

    1. f 1(x ) =14

    ln x2, x > 0

    2. f 1(x ) = (4ln x )2, x 13. f 1(x ) =

    14

    ln x2, x 1 correct

    4. f 1(x ) = (ln 4 x )2, x > 1

    5. f 1(x ) = ln x4

    2, x > 0

    6. f 1(x ) = (4ln x )2, x > 0

    Explanation:The restriction x > 0 on the domain of f is needed so that the term x is well-dened.But this puts a restriction on the range of f and hence on both the domain and range of the inverse f 1 of f . Indeed, since ex 1when x 0, we see that

    domain( f ) = [0 , ),range( f ) = [1 , ) ,

    which in turn means that

    domain( f 1) = [1 , ),range( f 1) = [0 , ) .

    Already this eliminates three of the answers,

    so we now need to determine f 1

    explicitlyFirst sety = e4 x

    and solve for x :

    x = 14

    ln y, i.e., x =14

    ln y2

    .

    Consequently, after interchanging x and we see that

    f 1(x ) =1

    4 ln x

    2, x

    1 .

    020 10.0 points

    Find the value of f (1) whenf (x ) = tan 1 x 4sin1 x .

    1. f (1) = 34

    2. f (1) = 94

    3. f (1) = 74

    correct

    4. f (1) = 11

    4

    5. f (1) = 54

    Explanation:Since

    tan 1(1) = 4

    , sin1(1) = 2

    ,

    we see that

    f (1) = 2 14

    = 74

    .

    021 10.0 points

  • 8/9/2019 HW02 Solutions-Calc408K UT

    11/11

    lin (ll25879) HW02 schultz (55325) 11

    Simplify the expression

    y = sin tan 1 x 3

    by writing it in algebraic form.

    1. y = x

    x 2 32. y =

    xx 2 + 3

    3. y = x x 2 + 3 correct

    4. y = 3

    x 2 + 3

    5. y =

    x 2 + 3 3

    Explanation:The given expression has the form y = sin

    where

    tan = x 3 ,

    2

    < < 2

    .

    To determine the value of sin given the valueof tan , we can apply Pythagoras theoremto the right triangle

    3

    x

    x 2 +

    3

    From this it follows that

    y = sin = x x 2 + 3 .

    Alternatively, we can use the trig identity

    csc2 = 1 + cot 2

    to determine sin .

    keywords: TrigFunc, TrigFuncExam,